New Free 120 (Step 2 CK)

These answer explanations are and always will be free. However, given multiple email requests, I will post my Venmo (@Adam-Zakaria-SLO) if you want to send a few dollars to show your support for the website.

I also offer reasonably priced Study Guides and Personalized Study schedules, so please reach out using the Tutoring menu option or Study Guides and Personalized Study schedules menu option listed above if you would like personalized support.

Furthermore, I offer personal statement and application review services for residency applicants, so please reach out using the “Residency Advising and Application Preparation” menu option above if interested.

Lastly, please check out my Youtube channel (https://www.youtube.com/channel/UCT1Ukl4pm5QK9iw6h4MB_Hw/playlists) and the “Biostatistics Curriculum” option above for free videos and practice questions reviewing all the essential biostatistics topics covered on NBME exams. Good luck with your exams!

Link to New Free 120 https://drive.google.com/file/d/1MMnrXDeVHeDLcjHmyPVdpkNUCHNiEkjY/view?usp=sharing

Block 1:

1) A 68-year old man is brought to the emergency department because of a 2-day history of progressive lethargy and tremors…

Oral lactulose therapy

  • Elderly man with a history of alcoholic cirrhosis who presents with altered mental status, tremors and asterixis (specific for hepatic or uremic encephalopathy), most consistent with hepatic encephalopathy
  • Key idea: Treatments for hepatic encephalopathy include lactulose (causes NH3 –> NH4+ which is trapped in gut and will not be absorbed) and rifaximin (kills gut bacteria –> less NH3 production)
  • While spontaneous bacterial peritonitis (along with other infections) could lead to hepatic encephalopathy, we have less concern for that diagnosis in this case because the patient is not febrile (which is >100F in a patient with cirrhosis) and has no abdominal pain/tenderness
  • Key idea: While the patient has a significant alcohol history, in this case we would not be as concerned about alcohol withdrawal symptoms (in which case lorazepam would be the answer) because he last had alcohol 2 weeks ago and the classic alcohol withdrawal symptoms typically occur within ~4 days (hallucinations and seizures occur around 24-48 hours since last drink, delirium tremens 48-96 hours since last drink)
  • Acute thiamine deficiency –> Wernicke encephalopathy –> Confusion, ataxia, ophthalmoplegia
  • Chronic thiamine deficiency –> Korsakoff –> Confabulation (patient is making up stories/lying but thinks that they are telling the truth), mental status changes

2) A 45-year-old woman comes to the emergency department because of a 3-day history of progressive shortness of breath, cough…

Serum antineutrophil cytoplasmic autoantibody assay (ANCA)

  • Young woman presenting with adult-onset asthma, hemoptysis, fever and sensory abnormalities of the upper and lower extremity most concerning for Eosinophilic granulomatosis with polyangiitis (Churgg-Strauss)
  • Key idea: Mnemonic for Churgg-Strauss is PAN (Palpable purpura, Asthma/sinusitis, Neurologic symptoms (wrist drop))
  • Key idea: In the setting of adult-onset asthma, important mimickers to consider include Churg-Strauss and allergic bronchopulmonary aspergillosis (often intermittent flares of respiratory symptoms in patient with cystic fibrosis and/or asthma)
  • Serum antiglomerular basement membrane assay –> Goodpasture syndrome –> Hemoptysis and hematuria often in a young man (no neurologic symptoms)
  • Determination of serum ACE activity –> Sarcoidosis –> Wide array of symptoms, but often will have respiratory symptoms, skin symptoms and bilateral hilar adenopathy

3) An 87-year-old woman, gravida 6, para 6, is brought to the emergency department because of a 24-hour history of severe…

Urinary retention

  • Elderly woman with many vaginal deliveries presenting with inability to void that resolves with catheterization and with physical exam most consistent with pelvic organ prolapse
  • Key idea: Patient is not having loss of urine, and therefore all of the incontinence answers must be incorrect
  • UTI would have suprapubic tenderness and would have an abnormal urinalysis (positive leukocyte esterase, positive nitrite, increased WBCs)

4) A 25-year-old woman is admitted to the hospital because of a 2-day history of fever, dizziness, light-headedness…

Methimazole

  • Young woman with multiple medication exposures presenting with signs of infection found to have neutropenia, most consistent with agranulocytosis due to methimazole
  • Key idea: Typically neutrophils make up ~65-70% of total WBCs, and we would expect that percentage to be even higher in the setting of an acute infection
  • Common causes of agranulocytosis include hyperthyroidism medications (methimazole, propylthiouracil), clozapine (antipsychotic medication) and carbamazepine (anti-epileptic drug)
  • Key idea: If a patient recently began treatment with methimazole/propylthiouracil and presents with fever and a sore throat, the next best step would be to obtain a CBC because that is a common presentation of agranulocytosis

5) A 16-year-old boy is brought to the emergency department because of a 2-day history of fever, nausea, vomiting, headache…

Antibiotic therapy

  • Adolescent boy with a history of splenectomy without antibiotic prophylaxis who presents with fever and other non-specific symptoms, most consistent with an infection and given the fact that the patient had a splenectomy we would be concerned about an encapsulated bacterial organism (SHiN: Strep pneumo, Haemophilus influenzae, Neisseria)
  • Key idea: In this case, our concern for a bacterial infection is high enough given the patient risk factors and presentation that we would begin broad-spectrum empiric antibiotics before ordering diagnostic tests
  • Key idea: Remember that patients with sickle cell disease often require prophylactic antibiotics because they have functional asplenia due to repeated vascular insults to the spleen

6) A previously healthy 29-year-old woman is admitted to the hospital because of a 1-day history of moderate headache, fever…

Adherence to immunization guidelines

  • Young college student with an unknown vaccination history with classic presentation for meningococcemia (fever, altered mental status, nuchal rigidity, diffuse purpuric rash)
  • Key idea: Meningococcemia is highly infectious and often spread in crowded areas (college dorms, summer camp, etc.), which is why individual receive a meningitis booster vaccine in adolescence
  • While isolating students suspected of meningitis would also be important, in this case if the patient had been fully immunized her risk would have been much lower
  • Wisdom tooth extraction is a risk factor for subacute endocarditis (fever, new murmur, potential embolic phenomenon) and Ludwig angina (submandibular cellulitis –> fever, drooling, stridor, respiratory distress, etc.)

7) A 27-year-old primigravid woman comes to the physician for her first prenatal visit 10 weeks after a home pregnancy test…

HIV

  • Young woman at 10 weeks pregnancy with normal pregnancy thus far who should receive routine prenatal testing
  • Routine testing at initial visit: HIV, Urine protein/culture, Chlamydia PCR, Rh(D) antibody screen, Hemoglobin/Hematocrit, Hepatitis B serum antigen, Rubella and varicella titers, Pap test (if indicated)
  • 24-28 weeks: Rh(D) antibody screen, gestational diabetes screen, hemoglobin/hematocrit
  • 35-37 weeks: Group B Strep culture

8) An investigator would like to decrease the incidence of type 2 diabetes mellitus among adult patients with risk factors…

Withdraw from the study now

  • Study participants are allowed to drop out whenever they want, regardless of the investigator’s desires
  • Similar concept to prioritizing patient choice/autonomy so long as it does not endanger the public or endanger a child (in the case of parents refusing life-saving therapy for their child)

9) A 70-year-old man comes to the clinic because of a 3-day history of fatigue, dark urine, decreased urine output, mild pain…

Measurement of postvoid residual volume

  • Elderly man presenting with low-grade fever, dark urine and decreased urine output with a otherwise normal exam concerning for an acute kidney injury
  • Key idea: In elderly men, a common cause of acute kidney injury is post-renal due to benign prostatic hyperplasia (BPH), which can be worked up with a bladder ultrasound
  • It would be important to discontinue NSAIDs in setting of suspected AKI, but acetaminophen should be okay
  • Oral TMP-SMX would be appropriate in setting of UTI or acute prostatitis, but it is not clear that he has either of those diagnoses

10) A 52-year-old woman comes to the physician because of a 2-day history of severe pain and markedly decrease range of motion in her…

Vancomycin

  • Middle aged woman with recent flare of rheumatoid arthritis who presents with fever and arthritis of the right elbow with a joint tap revealing a leukocyte count of 60K, most consistent with septic arthritis
  • Key idea: Synovial fluid gram stain only has a sensitivity of about 15%, so there are lots of false negatives
  • Key idea: Most common organism to cause septic arthritis is Staph aureus and patients with suspected Staph aureus infections are often started on an antibiotic that can cover MRSA, such as Vancomycin

11) A 52-year old man with hepatic cirrhosis comes to the emergency department because of a 3-hour history of vomiting blood…

5

  • ABSOLUTE risk reduction = (Risk of recurrent portal-systematic encephalopathy in patients who received EST) – (Risk of recurrent portal-systematic encephalopathy in patients who received EPCS) = 0.35 – 0.15 = 0.20
  • Number needed to treat = 1 / (Absolute risk reduction) = 1 / 0.20 = 5
  • Key idea: For number needed to treat calculation, only can use ABSOLUTE risk reduction and NOT relative risk reduction

12) Which of the following most strongly limits the generalizability of the study’s findings?

EPCS is available only at specialty centers

  • Internal validity = Are the study results accurate/correct for the population under study?
  • External validity (generalizability): Can the study findings be generalized/extended to other study groups, situations, etc.?
  • In this case, it is understood that EPCS is a highly specialized procedure and therefore the study results may not be generalizable to centers that are unable to perform EPCS
  • Allocation concealment –> technique used to PREVENT selection bias by concealing the allocation sequence until the moment of assignment
  • Although we prefer to have patients be blinded, in certain situations it is not possible especially when conducting an invasive procedure

13) Which of the following conclusions is most appropriate based on the results presented in the table?

EPCS is more effective than EST in decreasing hospital readmissions for variceal bleeding requiring transfusion

  • We can see that mean number of hospital readmissions for transfusion in setting of variceal bleeds was 6.8 in patients with EST and 0.4 in patients with EPCS with a p-value < 0.001, which is a statistically significant result given that it is below the conventional alpha value of 0.05
  • The 95% confidence interval for the difference in survival between EPCS and EST for Childs-Pugh Class A patients would NOT include 0 years because there was a statistically significant result for that result (p-value = 0.003), meaning that we have rejected the null hypothesis that there would be no difference in survival in that group
  • The authors did not compare survival between Childs-Pugh Group B and C, so while that statement may be true, we have no proof

14) A 42-year-old man comes to the office as a new patient. Three weeks ago, he was hospitalized for treatment of community-acquired…

Cardiovascular disease

  • Patients with HIV who are adherent to their medication regimen are at very low risk for an opportunistic infection or HIV-related malignancy
  • Key idea: Patients with HIV actually have accelerated cardiovascular disease, even among patients on antiretroviral therapy, and therefore given that cardiovascular disease is the #1 cause of death in the general public it is easy to imagine that it would also be the #1 cause of death in patients with HIV (https://www.ncbi.nlm.nih.gov/pmc/articles/PMC3964878/)

15) A 40-year-old divorced woman comes to the office because of a 1-year history of difficulty sleeping. She says…

Education on sleep hygiene

  • Poor sleeping habits displayed by patient: Sleeping late at night and smoking late at night (tobacco is a stimulant so basically equivalent of drinking coffee late at night)
  • Key idea: First step in many problems associated with insomnia or difficulty sleeping is to educate patient about proper sleep hygiene
  • General test-taking strategy: In the setting of a non-life threatening, non-progressive illness, conservative management is often the best first step (high blood pressure, high lipids, etc.)

16) A 58-year-old woman is brought to the office by her husband because of a 6-week history of increasing confusion…

Measurement of cerebrospinal fluid 14-3-3 protein concentration

  • Middle-aged woman presenting with subacute (weeks-months) of unprovoked altered mental status, loss of functional status and increased startle with otherwise normal vitals and labs, most consistent with Creutzfeldt-Jakob disease
  • Key idea: Rapidly progressive dementia should make you think of Creutzfeldt-Jakob disease and myoclonus or increased startle reflex fairly specific for Creutzfeldt-Jakob
  • Diagnostic findings that are specific for Creutzfeldt-Jakob disease: (1) Periodic sharp wave complexes on EEG (2) Increased 14-3-3 CSF assay (3) MRI findings on caudate nucleus and/or putamen
  • Increased CSF beta amyloid –> Alzheimer’s disease (older patient, develops over years)
  • Increased methylmalonic acid –> Vitamin B12 deficiency –> Megaloblastic anemia, neurologic symptoms
  • Serum protein electrophoresis –> Multiple myeloma –> Hypercalcemia, bone pain, anemia, kidney problems, etc.

17) A 38-year-old woman is brought to the emergency department by paramedics 90 minutes after she fell through…

Submerge the feet in a moderately warm whirlpool

  • Young woman who was submerged in ice water for 90 minutes brought to the ED with hypothermia and concern for frostbite of foot
  • Key idea: Frostbite should be managed by soaking the area in warm water (100-105 F) until it appears red and feels warm (https://www.redcross.org/content/dam/redcross/atg/PDF_s/Preparedness___Disaster_Recovery/Disaster_Preparedness/Winter_Storm/Frostbite_and_Hypothermia.pdf)
  • Key idea: Frostbite will initially be cold with a stinging sensation but will become red, blotchy and painful after rewarming
  • Can draw connection with management of heat stroke, where patients are often treated with rapid cooling by submerging in cold water

18) A 52-year-old woman comes to the emergency department because of a 1-day history of bleeding gums and…

Prednisone therapy

  • Young woman with lupus presenting with bleeding gums and petechiae/ecchymosis of extremities found to have mild anemia and pronounced thrombocytopenia most consistent with immune thrombocytopenia
  • 1st-line treatment of immune thrombocytopenia: Steroids, IVIG
  • 2nd-line treatment of immune thrombocytopenia: Rituximab, Splenectomy
  • Key idea: Patients with lupus tend to have pancytopenia, most commonly due to autoantibodies leading to immune-mediated destruction (particularly of platelets and leukocytes; multiple potential ways lupus can lead to anemia)
  • Key idea: Immune thrombocytopenia classically leads to increased number of megakaryocytes on bone marrow biopsy and few platelets on peripheral smear

19) A 13-year-old boy is brought to the emergency department because of a 3-day history of progressive fatigue, shortness of breath…

Myocarditis

  • Adolescent boy with recent viral illness presenting with fatigue and respiratory distress found on physical exam to have hypotension, lung crackles, and a new cardiac murmur most consistent with myocarditis
  • Key idea: Most common cause of acute heart failure in young people is myocarditis, which will almost always follow a viral URI (especially on the NBME)
  • Key idea: While Strep infection –> Rheumatic fever –> Valvular dysfunction –> Heart problems, it would NOT develop over 1 week (and rheumatic fever will virtually only be seen in immigrants from non-Western countries on NBME)

20) A 10-day-old female newborn is brought to the emergency department because of a 2-day history of bilious vomiting…

Volvulus

  • 10 day old newborn who was healthy at birth but now is presenting with bilious vomiting, abdominal distention and abdominal tenderness concerning for malrotation c/b volvulus
  • Key idea: Other common causes of bilious vomiting in a newborn would include duodenal atresia or etiologies of failure to pass meconium (meconium ileus, Hirschsprung disease), but the fact that the patient was normal in the first week of life means that these conditions would be highly unlikely
  • Intussusception –> Toddler who presents with recurrent episodic abdominal pain +/- bloody stools that improves when drawing knees to chest
  • Necrotizing enterocolitis –> Sepsis, hypotension, bloody stool, abdominal pain in a newborn born prematurely of with a congenital heart problem

21) An 85-year-old man with severe dementia, Alzheimer type, is admitted to the hospital because of a nonhealing ulcer on…

Absence of resource stewardship

  • Resource stewardship: Appropriate allocation of resources that leads to high-value, evidence-based care
  • Key idea: The patient has been bedridden for 3 years, so using an expensive, long-lasting stent to re-establish blood flow is likely not going to lead to any gains in function (i.e. use of the expensive stents is not high-value care)
  • Although ageism and health care disparities are important public policy considerations, in this case the patient was able to receive the expensive treatments so it doesn’t apply

22) A 62-year-old woman with myasthenia gravis comes to the office because of a 2-week history of falling…

Assessment of medical decision-making capacity

  • A middle-aged woman with myasthenia gravis who has stopped using her pyridostigmine and now has very symptomatic disease with high risk of respiratory compromise who is refusing hospitalization and therefore should have their decision-making capacity checked
  • Key idea: Patient autonomy is often valued over other tenants of medical ethics, but key to this point is that the patient has capacity to make medical decisions
  • Aspects of medical decision-making capacity: Communicating a choice, Demonstrating understanding of risks/benefits, providing rationale for decision, Consistent decision

23) A 55-year-old man is brought to the emergency department by paramedics 45 minutes after his daughter found him unresponsive…

Vitamin B1 (thiamine)

  • Middle-aged patient with chronic history of alcoholism brought to the ED due to altered mental status found to be hypotensive, hypothermic, hypoglycemia and urine toxicology screen positive for alcohol and benzodiazepines
  • Key idea: On the test ALCOHOLIC = MALNOURISHED!!!
  • Key idea: Many patients with history of alcoholism presenting with altered mental status are started on a “banana bag(thiamine, folate, magnesium, multivitamin)
  • Key idea: Thiamine should always be given before glucose because if you give glucose first you can theoretically push a patient into Wernicke encephalopathy (confusion, ataxia, ophthalmoplegia) [although some research suggests this may just be a myth]
  • If patient had altered mental status, pinpoint pupils, respiratory depression –> Concern for opiate intoxication –> Naloxone

24) A 16-year-old boy is brought to the emergency department because of a 6-hour history of severe right-sided scrotal pain…

Testicular torsion

  • Adolescent boy with acute, unilateral testicular pain with physical exam demonstrating elevated right testicle with absent cremasteric reflex on the right with a negative Prehn’s sign (pain unrelieved with elevation) most consistent with testicular torsion
  • Testicular torsion leads to loss of cremasteric reflex because the reflex is mediated by the genitofemoral nerve, which travels within the spermatic cord and is thereby affected by torsion
  • Key idea: If question asked about next step, it would be go straight to OR and need perform orchiopexy on both testicles (because other testicle will also be at risk for torsion)
  • Testicular pain relieved with elevation –> Positive Prehn’s sign –> Epididymitis

25) A 5-year-old boy is brought to the emergency department because of a 5-day history of right-sided facial droop…

Discuss the family’s goals for the patient

  • Child presenting with new neurologic symptoms found to have brain mass that apparently has a poor prognosis –> Next best step is determine the patient and family’s goals to guide care

26) A 50-year-old woman comes to the office because of a 3-month history of a facial rash that has increased in size…

Sarcoidosis

  • Middle aged African-American woman presenting with subacute raised, violaceous facial lesions consistent with lupus pernio and shortness of breath with bilateral hilar lymphadenopathy on imaging, most consistent with sarcoidosis
  • Other potential symptoms of sarcoidosis: Bell’s palsy, uveitis, erythema nodosum
  • Differential of erythema nodosum: Sarcoidosis, Inflammatory bowel disease, tuberculosis, Behcet’s syndrome, fungal infections (cocci, histo, etc.)

27) A 64-year-old woman comes to the clinic because of a 2-week history of severe pain and generalized muscle weakness..

Serum creatinine kinase

  • Middle-aged man who recently started a statin and presents with subacute muscle pain and weakness with otherwise normal exam most consistent with statin-induced myopathy (leads to increased CK and normal ESR)
  • Differential of myopathy (bilateral, symmetric, proximal muscle weakness): (1) Glucocorticoid-induced: Signs of Cushing’s (abdominal obesity, buffalo hump, etc.) with normal ESR and normal CK (2) Statin-induced: Muscle pain and weakness with increased CK and normal ESR (3) Hypothyroid: Features of hypothyroidism (weight gain, cold intolerance, etc.) with normal ESR and increased CK (4) Polymyositis/Dermatomyositis: Inflammed with increased ESR and increased CK (5) Polymyalgia rheumatica: Stiffness, associated with temporal arteritis, increased ESR and normal CK

28) A 23-year-old man who is on active duty in the US Army comes to the clinic 1 hour after sustaining a bee sting to this right cheek…

Cleanse the site and apply ice

  • Young otherwise healthy man who had a bee sting on his right cheek with no signs concerning for anaphylaxis or signs of eye problems (vision problems, ophthalmoplegia, etc.) who should therefore be treated conservatively

29) A 32-year-old man is brought to the emergency department after being found by his wife in their bedroom with a gun in his hands…

Electroconvulsive therapy

  • Young man with acute life stressor with signs of depression (tearful affect, sleep problems, loss of functional status, etc.) + risk of suicidality (found in bedroom with gun) + Psychotic features + Refusal to eat/drink who should therefore be treated with electroconvulsive therapy
  • Indications for electroconvulsive therapy in setting of depression: (1) Refractory to standard treatment (2) Presence of psychotic features (3) Psychiatric emergency (pregnancy patient, refusal to eat/drink, imminent risk of suicide)
  • Key idea: No absolute contraindications to electroconvulsive therapy
  • Transcranial magnetic stimulation used in Parkinson’s
  • Mirtazapine is a good antidepressant in people who are not eating enough and who have trouble sleeping

30) A 60-year-old woman comes to the office as a new patient. She recently moved to the area. She feels well. She has…

Hepatitis C

  • USPSTF previously recommended a one-time screening for Hepatitis C among adults born between 1945 and 1965, but now recommends one-time screening for Hepatitis C among all adults 18 to 79 years old
  • Most conservative breast cancer screening schedule is every year starting at 40 years of age (least conservative would be every 2 years starting at 50), and this patient had mammogram 6 months ago
  • AAA screening (abdominal ultrasound) done in 65-75 year old MEN with ANY history of smoking (no 30 pack-year cut-off like for low-dose CT screenings for lung cancer)
  • Between 30 and 65 years of age woman should have Pap smear every 3 years or Pap+HPV (co-testing) every 5 years
  • Starting at 50 (or now 45!) patients should have a colonoscopy every 10 years (assuming normal screens and no increased risk like IBD or family member with early colon cancer)
  • USPSTF recommends chlamydia screening in all sexually active women 24 years or younger (even if they are monogamous!) but recommends AGAINST screening women 25 years or older if they are not at increased risk due to sexual practices

31) A 12-year-old girl is brought to the physician for a follow-up examination 8 days after starting a course of oral penicillin…

Use of a nonpenicillin antibiotic to treat the pharyngitis

  • A young girl who was started on a penicillin for strep pharyngitis 8 days ago presenting with fever, lymphadenopathy, urticaria, arthralgias and proteinuria most consistent with serum sickness
  • Key idea: Classic Type III sensitivity, meaning that pathology is mediated by deposition of antigen-antibody complexes –> to prevent future episodes we want to avoid giving patient “antigen” and therefore we should avoid penicillins
  • Key idea: Serum sickness often triggered by certain drugs (antibiotics) or infections (hepatitis B)

32) A 45-year-old woman is hospitalized for management of Staphylococcus aureus endocarditis with persistent bacteremia…

Encourage questions from all team members

  • Question asks which strategy will improve communication during rounds, not necessarily what would improve patient safety or efficiency of rounds

33) An 82-year-old man comes to the physician 1 month after he noticed a rough, painless lesion on his right hand. The lesion…

Keratoacanthoma

  • Elderly man with significant sun exposure (farmer) who presents with a progressive 2 cm raised skin lesion with central keratin debris most consistent with a keratoacanthoma (variant of squamous cell skin cancer)
  • Key idea: Often keratoacanthomas will grow for several months and then regress spontaneously, but they can often be treated with excision to be safe and rule out invasive cancer
  • https://dermnetnz.org/topics/keratoacanthoma/

34) A 37-year-old man comes to the physician for a routine health maintenance examination. He has no history of

Serum lipid studies

  • Nearly 40 year old man comes in for regular primary care appointment who is borderline obese and nearly hypertensive, and therefore should have regular screening for chronic conditions such as hyperlipidemia (lipid studies) and diabetes (glucose, HbA1c)
  • Key idea: USPSTF strongly recommends screening men over 35 (among other groups) for lipid disorders (https://www.uspreventiveservicestaskforce.org/uspstf/recommendation/lipid-disorders-in-adults-cholesterol-dyslipidemia-screening-2008)

35) A 35-year-old woman is evaluated for a persistent fever 4 days after admission to the hospital for treatment of pyelonephritis…

CT scan of the abdomen

  • Young woman with poorly controlled type 2 diabetes currently admitted for pyelonephritis that is refractory to standard treatment (fluoroquinolone) with persistent fevers and costovertebral tenderness, concerning for potential complicated pyelonephritis that should be worked up with symptoms
  • Key idea: Most cases of pyelonephritis do not require imaging and the diagnosis can often be made with clinical presentation + urine studies + blood culture
  • Indications for imaging (CT abdomen/pelvis) in patient with pyelonephritis: (1) Persistent symptoms despite 48-72 hours of therapy (2) History of nephrolithiasis (3) Unusual urinary findings (gross hematuria) (4) Concern for complicated pyelonephritis (renal abscess, emphysematous pyelonephritis, septic shock)
  • Likely would want to wait on blood cultures before deciding to broaden coverage because quinolones should cover almost all causative pathogens in setting of pyelonephritis

36) A 23-year-old man is being evaluated in the intensive care unit because of steadily decreasing blood pressure since he…

Begin a continuous infusion of norepinephrine

  • Young man with significant immunodeficiency admitted to ICU with septic shock and hypotension refractory to 4L of fluid showing early signs of volume overload (inspiratory crackles, non-collapsible IVC) who should therefore be started on norepinephrine for blood pressure support rather than given more fluids
  • Key idea: Norepinephrine is very useful in hypotension due to septic shock due to primarily alpha-1 effects, whereas epinephrine is more useful in anaphylaxis due to its beta-2 effects leading to bronchodilation (https://pubmed.ncbi.nlm.nih.gov/28509668/)
  • IV hydrocortisone would be more useful in setting of acute hypotension due to adrenal insufficiency
  • Increased intracranial pressure would lead to hypertension with bradycardia, whereas this patient has hypotension and tachycardia

37) A 32-year-old nulligravid woman comes to the office for counseling prior to conception. Menses occur at irregular…

Gestational diabetes

  • Young nulligravid woman with impaired glucose tolerance at baseline which would put her at increased risk for gestational diabetes
  • Key idea: Pregnancy can lead to gestational diabetes because the placenta releases human placental lactogen (HPL), which leads to increased insulin resistance such that there will be more blood sugar in the bloodstream –> more sugar delivered to fetus
  • Patients with gestational diabetes tend to have macrosomic babies
  • Main risk factor for placenta previa is previous C-section (abnormal uterus wall –> implantation of placenta over cervix)
  • Main risk factor for pre-term labor include previous history of pre-term labor and short cervix

38) A 17-year-old girl is brought to the physician by her parents because they are concerned she is using illicit drugs…

Inform the parents that the physician cannot deceive the physician

  • Key idea: Active deception of patients is generally frowned upon, and in an adolescent they have confidentiality/autonomy around sex (contraception), drugs (rehab programs, drug screening) and rock and roll (emergency situations)
  • Key idea: Everything the patient tells the physician is NOT confidential because if the patient is at imminent risk of harming themselves or others than that is generally not confidential and in a non-adult they would still require parental consent for most procedures/interventions (except for those things listed above)

39) A 52-year-old woman with type 2 diabetes mellitus comes to the office for a follow-up examination. Two weeks ago…

Acute kidney injury

  • Middle-aged woman with type 2 diabetes and hypertension recently started on lisinopril who presents with signs of volume overload (lower extremity edema, lung crackles, S3 gallop), worsening hypertension and a severely increased creatinine (1.2 –> 4.0) most consistent with some form of acute kidney injury
  • Key idea: Often if a patient develops an AKI after starting lisinopril, it is a sign of poor blood flow to the kidneys (renal artery atherosclerosis, fibromuscular dysplasia) because in those patients angiotensin II-mediated constriction of the efferent arteriole, while damaging to the kidney long-term, enables maintenance of an appropriate GFR and abrupt removal of that activity through use of an ACE inhibitor can lead to an AKI (pre-renal –> Acute tubular necrosis)
  • Patient has increased intravascular volume with signs of decreased effective circulating volume (similar to patient with decompensated heart failure, cirrhosis, etc.)
  • Acute glomerulonephritis –> Hematuria, Dysmorphic RBCs, RBC casts
  • Acute interstitial nephritis –> Fever, rash, eosinophils in urine, WBC casts

40) A 15-year-old girl is brought to the emergency department by her father because of a 2-hour history of right leg pain that began…

Osteogenesis imperfecta

  • Adolescent girl with history of hearing loss, multiple fractures, and hyperflexible joints presenting with potential lower extremity fracture in setting of minor trauma, most concerning for osteogenesis imperfecta
  • Signs of osteogenesis imperfecta: (1) Frequent fractures (2) Blue sclerae (due to ability to see underlying choroidal veins) (3) Conductive hearing loss (4) Short-normal stature (5) Dentinogenesis imperfecta (thin, greyish teeth) (6) Joint hypermobility
  • Osteoporosis –> Decreased bone density +/- fragility fractures in an at-risk patient (old, long-term steroid use, decreased weight-bearing exercise, etc.)
  • Paget disease –> Disordered bone growth which is often asymptomatic, but patients can have bone pain, hearing loss, headache, spinal stenosis
  • Osteomalacia/Rickets –> Poorly mineralized bone often in setting of vitamin D deficiency or problems activating vitamin D (liver/renal disease) which often leads to wrist widening, frontal bossing, leg bowing, hypertrophy of costochondral joints (“rachitic rosary”) , etc.

Block 2:

1) A 22-year-old primigravid woman at 10 weeks’ gestation comes to the emergency department (ED) because of a 3-day history…

How do you think he will react?

  • Key idea: In these communication questions, you generally want to pick the most open-ended question (i.e. question that CANNOT be answered yes/no)
  • All of the other questions are presumptuous; best to first allow the patient to clarify their concerns in their own words

2) Five days after admission to the hospital for management of an ST-elevation myocardial infarction, a 59-year-old man…

Aortic embolism

  • Middle-aged man with ST-elevation MI complicated by extensive anterolateral akinesis who on day 5 of hospitalization develops bilateral lower extremity pain/paresthesias, cyanosis, hypotension, and weak upper extremity pulses with absent lower extremity pulses, most consistent with aortic embolism
  • Aortic dissection –> Tearing chest pain radiating to the back, asymmetric pulses, risk factors of chronic HTN or cocaine/methamphetamine
  • Pericardial tamponade –> Hypotension, elevated JVD, distant heart sounds, electrical alternans
  • RV failure –> Lower extremity edema, elevated JVD, hepatomegaly, NO LUNG CRACKLES (left heart function preserved)

3) A 75-year-old man comes to the clinic because of a 2-year history of progressive shortness of breath. He also…

High-resolution CT scan of the chest

  • An elderly man with a significant smoking history presents with chronic shortness of breath and cough with physical exam consistent with hypoxemia, fine/dry crackles of both lungs and clubbing with chest x-ray showing scattered reticular opacities most consistent with idiopathic pulmonary fibrosis –> Needs to be worked up with a high-resolution CT
  • Key idea: Normal CT better for detecting cancer, whereas high-resolution CT needed to have enough resolution to see the fibrotic lung disease
  • Key idea: IPF exclusively seen in older patients and much higher risk of IPF in patients with significant smoking history (https://www.nhlbi.nih.gov/health-topics/idiopathic-pulmonary-fibrosis)
  • Key idea: On NBME exam, crackles often mean (1) Pulmonary edema (heart failure exacerbation, etc.) (2) Interstitial lung disease (IPF, sarcoidosis)
  • Lung biopsy –> Highly invasive test which should likely be preceded by imaging and/or other diagnostic tests
  • Thoracic PET scan –> Lung cancer (weight loss, fatigue, metastatic/paraneoplastic complications, etc.)
  • Serum antiglomerular basement membrane antibody assay –> Goodpasture syndrome –> Young man with hemoptysis and hematuria

4) A previously healthy 20-year-old woman, who is a sophomore in college, is brought to the emergency department by her roommates…

K+: Decreased // Cl-: Decreased // HCO3-: Increased

  • Young woman with history compatible with bulimia nervosa (normal weight individual with history of binge eating followed by compensatory vomiting) who presents with symptoms concerning for Mallory-Weiss syndrome (vomiting blood) and signs of intravascular volume depletion (dizziness, low blood pressure, tachycardic, etc.)
  • Vomiting or Nasogastric tube suction –> Loss of HCl (stomach acid) –> Low H+ (metabolic alkalosis) and Low Cl (hypochloremia)
  • Vomiting or Nasogastric tube suction –> Intravascular volume depletion –> Activation of the Renin-Angiotensin-Aldosterone system (RAAS) –> Increased aldosterone –> Stimulation of ENaC channels –> Increased Na+ reabsorption with compensatory increased secretion of K+ and H+ to maintain electrical neutrality –> Low K+ (Hypokalemia) and Low H+ (Metabolic alkalosis)

5) A 58-year-old man comes to the clinic for a follow-up examination 3 months after beginning warfarin therapy for deep venous…

Cystoscopy

  • Middle-aged man with significant smoking history who presents with hematuria with signs of benign prostatic hyperplasia (BPH) and labs demonstrating gross hematuria with no signs of glomerulonephritis (no RBC casts, no dysmorphic RBCs, very little protein) or infection (negative nitrite and leukocyte esterase), leading to concern for bladder cancer which should be worked up with cystoscopy
  • Key idea: Bladder cancer, kidney cancer, and pancreatic cancer will almost always be seen in smokers on the NBME!
  • Risk factors for bladder cancer: Cigarette smoking, cyclophosphamide, occupational exposures (painters, metal workers)
  • Indications for cystoscopy: (1) Gross hematuria with no evidence of glomerular disease or infection (2) Obstructive symptoms with suspicion for stricture or stone (3) Recurrent UTIs (4) Microscopic hematuria in a patient with increased risk for malignancy
  • Note: INR between 2.0 and 3.0 is therapeutic for patients on Warfarin

6) A 32-year-old woman is brought to the emergency department immediately after being involved in a motor vehicle collision…

Operative reduction and internal fixation

  • Femoral fractures are VERY SERIOUS and are almost always treated with operative reduction and internal fixation +/- Prosthetic femoral head +/- intramedullary rod fixation
  • Key idea: Fracture of the femoral neck puts patients at risk for avascular necrosis of the femur
  • Key idea: Femoral shaft fractures that are OPEN (break the skin) are an orthopedic EMERGENCY requiring OR cleaning and closure

7) A 57-year-old man comes to the physician 1 week after he had a 45-minute episode of slurred speech and clumsiness of his right hand…

Antihypertensive therapy

  • Middle-aged man with significant hypertension, hyperlipidemia, and smoking history presenting 1 week after transient slurred speech and clumsiness of right hand, most consistent with a lacunar infarct (dysarthria-clumsy hand syndrome)
  • Key idea: The most important risk factor for lacunar infarcts and strokes is poorly controlled hypertension (remember that the common carotid is one of the first branches off the aorta, so if blood is being ejected with high amounts of force –> shearing of carotid –> atherosclerosis –> Stroke)
  • Key idea: For atherosclerotic disease (coronary artery disease, peripheral vascular disease, etc.) in a patient with every risk factor under the sun, the most important one will be smoking > diabetes
  • Four main types of lacunar infarct (highly associated with poorly controlled hypertension): (1) Pure motor stroke (2) Pure sensory stroke (3) Ataxia-hemiplegia syndrome (4) Dysarthria-clumsy hand syndrome

8) A 27-year-old primigravid woman at 21 weeks’ gestation comes to the emergency department because of a 2-day history of…

Influenza A virus

  • Young primigravid woman at 21 weeks’ gestation who presents with acute shortness of breath, GI symptoms, headache, muscle aches, fever and chest x-ray with unilateral interstitial infiltrates most concerning for influenza infection (“the flu”)
  • Key idea: Muscle aches and GI symptoms very common in patients with influenza infection and often can help differentiate it from a rhinovirus infection or a bacterial pneumonia
  • Key idea: While Strep pneumo is the most common cause of pneumonia, it would lead to a lobar consolidation on chest x-ray
  • Pseudomonas aeruginosa pneumonia –> Cystic fibrosis patients
  • H. influenzae pneumonia –> Patients with splenic dysfunction (encapsulated bacteria), often smokers
  • Legionella pneumonia –> Often smokers, accompanied by hyponatremia, diarrhea and neuro symptoms

9) A 7-year-old boy is brought to the physician because of a 1-year history of poor performance in school. His parents…

Learning disorder

  • Learning disorder: Child has a problem acquiring and applying skills in reading, writing or math, such as this child who has problems sounding out (reading) unfamiliar words
  • Importantly learning disorder =/= Intellectual disability and many individuals with learning disorders have average or above average intelligence
  • Expressive language disorder: PERSISTENT difficulties in comprehension and/or production of spoken and written language (this child has appropriate fluency and comprehension for age, he just struggles with new tasks)

10) A 23-year-old woman who is an active-duty enlisted airman in the US Air Force comes to the base clinic because of a 4-week history…

Otitis externa

  • Young woman with chronic earplug use who presents with subacute unilateral ear itchiness with physical exam notable for ear pain with manipulation of pinna and an erythematous/edematous ear canal most consistent with otitis externa
  • Key idea: Ear pain with manipulation of pinna is very specific for otitis externa, especially on the NBME
  • Key risk factors for otitis externa are trauma (ear plugs, etc.) and water exposure, and is often treated with ciprofloxacin ear drops

11) A 34-year-old woman who is on active duty in the US Army comes to the military urgent care clinic because of a 4-day history of headache…

Ibuprofen and decongestant therapy

  • Young woman presenting with 4-day history of headache, purulent nasal discharge and left facial pain without fever most consistent with viral sinusitis that should be managed conservatively (NSAIDs and decongestants)
  • Key idea: Most cases of sinusitis are viral in origin, but you should be concerned for bacterial rhinosinusitis if at least 1 of the following 3 criteria are met: (1) Persistent symptoms >10 days (2) Febrile illness for >3 days (3) Symptoms improve and then get worse
  • Bacterial rhinosinusitis –> Treat with amoxicillin + Clavulonic acid (Augmentin)

12) A 16-year-old boy with VATER syndrome is brought to the physician because of 2-week history of harsh cough that began with…

Tracheal collapse

  • Adolescent boy with VACTERL presenting with subacute cough accompanied by expiratory stridor, forced vital capacity > slow vital capacity and a scooped-out flow-volume loop, all of which are associated with tracheal collapse
  • Key idea: FORCED vital capacity (amount of air that can be FORCEfully expired) is almost always about equal to SLOW vital capacity (amount of air expired in passive breathe), with one exception being tracheal collapse because forceful expiration leads to obstruction –> less air expired
  • Scooped out flow-volume loop = Obstructive lung disease (problem getting air OUT of the lungs)
  • VATER syndrome = VACTERL = Constellation of mesodermal anomalies leading to Vertebral defects, Anal defects, Cardiac defects, Tracheal or Esophageal anomalies (often fistula), Renal defects and Limb defects
https://respiratorycram.com/flow-volume-loops/

13) A 44-year-old man comes to the emergency department 6 hours after the onset of increasingly severe substernal chest pain…

Echocardiography

  • Young man with recent viral illness (chills, fatigue, muscle aches) presenting to ED with POSITIONAL chest pain, fever, pericardial friction rub (scratchy sound), leukocytosis and an ECG with diffuse ST elevations and PR depressions most consistent with post-viral pericarditis
  • Key idea: Next best diagnostic step is an echocardiogram to both confirm the diagnosis and look at how much fluid is around the heart (concern for potential tamponade as complication)
  • Key idea: Cardiac ischemia would be non-positional with ECG changes in a specific region of heart (II, III, avF (inferior) /// V1-V4 (anterior) /// I, avL, V5-V6 (lateral)) and patient would have increased troponin

14) Echocardiography shows a small pericardial effusion with no tamponade. On examination an hour later…

Ibuprofen and Colchicine

  • 1st-line therapy for viral/idiopathic pericarditis: NSAIDs + Colchicine
  • 2nd-line therapy for viral/idiopathic pericarditis: Corticosteroids
  • Key idea: Uremic pericarditis should be treated with hemodialysis (AEIOU mnemonic to remember indications for hemodialysis)
  • Key idea: Peri-infarction pericarditis (several days after MI) should receive supportive management and NOT be treated with NSAIDS due to concern for impaired wound healing and possible increased risk of post-MI mechanical complications (free wall rupture, etc.)

15) A chief medical officer at a hospital has received several reports that one of her colleagues, a 64-year-old male surgeon…

Instruct the surgeon to take a temporary leave of absence and undergo a medical evaluation

  • Key idea: Doctors are human beings and often do not like to admit that there is a problem, so in this case the hospital leadership needs to enforce that the surgeon receive a medical evaluation and be medically cleared before being allowed to practice both for the benefit of the surgeon and, more importantly, for the safety of the patients

16) A 33-year-old woman is admitted to the hospital because of visual field loss following a 48-hour history of eye pain…

MRI of the brain

  • Young woman with neurologic symptoms separated by time and place presenting with optic neuritis, most consistent with multiple sclerosis
  • Key idea: While diagnosis of multiple sclerosis can be made either with MRI or lumbar puncture, MRI should be preferred over lumbar puncture because it is much less invasive
  • Key idea: Painful eye movements in a young woman should ALWAYS make you concerned for optic neuritis in the setting of multiple sclerosis
  • Key idea: Other MS-related eye condition is internuclear ophthalmoplagia (problem abducting one of the eyes)

17) A 68-year-old woman comes to the office because she is concerned about developing Alzheimer disease. She says she has…

No additional diagnostic studies are indicated

  • Older woman with mild memory problems but otherwise normal functioning most consistent with normal aging
  • Key idea: Alzheimer disease can only be diagnosed at autopsy, and therefore in a patient presenting with dementia Alzheimer disease should be treated as a diagnosis of exclusion and your goal in the work-up is to rule out potentially reversible causes of dementia (normal pressure hydrocephalus, B12 deficiency, depression, hypothyroidism, Neurosyphilis, etc.)
  • Normal aging: MOCA of 26 or higher, Normal functioning in activities of daily living
  • Mild cognitive impairment: Normal functioning in activities of daily living with compensation
  • Dementia: Significant decline in at least 1 cognitive domain, marked functional impairment, develops over years

18) A 55-year-old man with major depressive disorder is brought to the emergency department 1 1/2 hours…

Discontinue paroxetine and duloxetine

  • Middle-aged man is brought to the ED after potentially taking high doses of paroxetine (SSRI) and Duloxetine (SSNRI) together and presents with Agitation, increased neuromuscular Activity (increased deep tendon reflexes, myoclonus) and Autonomic hyperactivity (febrile, tachycardic, diaphoretic, mydriasis), which fits with the 3 A’s of Serotonin syndrome
  • Key idea: Serotonin syndrome often treated by discontinuing offending medication(s) +/- Cyproheptadine (serotonin receptor antagonist)
  • Important to contrast with neuroleptic malignant syndrome which is often seen in patients taking antipsychotic medications who present with Malignant FEVER (Myoglobinuria, Fever, Encephalopathy, Vitals unstable (autonomic hyperactivity), Enzymes elevated and Rigidity) and would be treated with Dantrolene
  • Key idea: Main differences between Serotonin syndrome and Neuroleptic malignant syndrome are inciting drug and presence of muscle breakdown (in NMS)

19) A 32-year-old man comes to the office for a follow-up examination 2 months after he was diagnosed with hypertension…

Determine whether the patient has had thoughts of harming others

  • Young man presenting with signs of psychosis (paranoid delusions) in setting of chronic history of similar complaints with an otherwise unremarkable physical exam and lab work-up concerning for schizophrenia or another primary psychotic disorder
  • Key idea: Although patient could likely benefit from antipsychotic medication (risperidone), main morbidity/mortality associated with schizophrenia and other psychotic disorder is if voices tell patient to hurt themselves or to hurt others, so this patient should first be screened for suicidal/homicidal ideation or intent

20) A 40-year-old woman is admitted to the hospital 12 hours after the onset of mild right upper abdominal pain and a pruritic…

Acyclovir

  • Young woman on immunosuppressants in setting of renal transplant who presents with fever, diffuse vesicular rash and RUQ tenderness with mildly elevated LFTs most concerning for disseminated zoster, which needs to be treated with IV acyclovir until all lesions are crusted
  • Key idea: While most cases of herpes zoster will be confined to a single dermatome, if a patient with zoster has >20 vesicles outside of primary dermatome, then they qualify as having disseminated zoster
  • Key idea: Patients with disseminated zoster need to be on airborne/contact precautions and should receive a chest x-ray and testing of LFTs because it most commonly involves the lungs and liver
  • Folliculitis –> Pustules WITHOUT FEVER OR SYSTEMIC FEATURES –> Dicloxacillin, cephalexin, etc.

21) An 87-year-old woman, gravida 2, para 2, is brought to the emergency department from a skilled nursing care facility because…

Vulvar carcinoma

  • Elderly woman with past cervical dysplasia presents with large, bleeding verrucous lesion of perineum most concerning for vulvar carcinoma
  • Key idea: Cervical cancer, Vulvar cancer, vaginal cancer, anal cancer and penile cancer are all typically mediated by HPV (consistent with verrucous-nature of lesion), with this patient having past history of HPV infection given cervical dysplasia
  • The lesion is NOT coming out of the anus, so hemorrhoids, rectal cancer and rectal polyps are ruled out

22) A 47-year-old woman comes to the office for a routine health maintenance examination. She feels well and has…

Ultrasonography of the thyroid gland

  • Key idea: Patients presenting with a thyroid nodule should ALL receive thyroid ultrasound and thyroid function tests (TSH, T3/T4, etc.)
  • Key idea: Hypofunctional (“cold”) nodule (normal-high TSH) MORE concerning for thyroid cancer
  • Indications for Fine-Needle biopsy: Thyroid nodule greater than 2 cm in diameter or thyroid nodule greater than 1 cm with high-risk ultrasound features (microcalcifications, irregular markings, etc.)
  • Ultrasound is the best imaging modality in evaluating the thyroid

23) A 1-hour-old male newborn is examined in the hospital nursery. He was born at term to a 22-year-old woman, gravida 1, para 1, with HIV infection…

Zidovudine (AZT)

  • Baby born to mother with well-controlled HIV (viral load < 1000 copies/mL): Zidovudine only
  • Baby born to mother with poorly controlled HIV (viral load > 1000 copies/mL): Multi-drug antiretroviral therapy
  • Key idea: Babies born to a mother with HIV should be tested for HIV 24 hours after birth and should NOT breastfeed

24) A 25-year-old woman comes to the office because she has not had a menstrual period for 6 months. Previously, menses…

Cabergoline therapy

  • Young woman presenting with anovulation, bitemporal hemianopsia, and galactorrhea found to have increased prolactin, normal thyroid function and 1.2 cm x 1.5 cm pituitary mass most consistent with a prolactinoma
  • Asymptomatic microprolactinoma (<1 cm): No treatment
  • Macroprolactinoma (>1 cm) or symptomatic microprolactinoma: Dopamine agonist (cabergoline, bromocriptine)
  • Indications for Surgical resection: Very large tumor (>3 cm) or tumor increases in size while using dopamine agonist
  • Key idea: Hypothyroidism –> Increased TRH –> Stimulation of prolactin release –> Increased prolactin, so when you see an elevated prolactin it is important to get thyroid function tests to rule out hypothyroidism leading to elevated prolactin

25) A 2-week-old boy is evaluated in the neonatal intensive care unit because of temperature instability and decreased activity…

X-ray of the abdomen

  • Premature infant (30 weeks gestation) who at 2 weeks of age has developed hypothermia, decreased activity, hypotension, abdominal tenderness, bloody stools and a leukocytosis with a neutrophilic predominance, most concerning for necrotizing enterocolitis
  • Best diagnostic step in necrotizing enterocolitis is X-ray of the abdomen which will show pneumatosis intestinalis (2 below, air in the bowel wall), portal venous gas (1 below, air in the portal vein) and/or pneumoperitoneum (air under the diaphragm)
  • Key idea: Necrotizing enterocolitis classically seen in premature babies or babies with congenital heart disease
  • Measurement of PT and aPTT: Vitamin K deficiency (bleeding into the ventricles of brain –> Decreased activity and altered mental status), DIC (bleeding from multiple orifices)
  • Measurement of stool reducing substances: Lactose or fructose metabolism problems
https://ep.bmj.com/content/105/1/50

26) A 13-month-old boy is brought to a small rural emergency department (ED) near his family’s community because of an erythematous…

The suburban hospital violated the Emergency Medical Treatment and Active Labor Act (EMTALA)

  • Hospital with a pediatric surgeon would DEFINITELY have ability to drain a perianal abscess and it is clear that they transferred the patient due to inability to pay
  • EMTALA: Emergency departments that accept payments from Medicare must provide an appropriate examination/care to anyone seeking care regardless of ability to pay, legal status or citizenship status (https://en.wikipedia.org/wiki/Emergency_Medical_Treatment_and_Active_Labor_Act)

27) A 25-year-old woman is brought to the emergency department by paramedics because of a seizure that began 45 minutes…

Video EEG monitoring

  • Young woman with past history of well-controlled epilepsy brought to ED because of episode concerning for pseudoseizure (no post-ictal state, no tongue biting, no incontinence, occurred following negative emotional state)
  • Key idea: Best test to CONFIRM diagnosis of pseudoseizure is Video EEG monitoring because you can look at whether the patient’s seizure-like symptoms correlate with EEG changes seen during a true seizure
  • Measuring antiepileptic concentration, neuropsych testing, and measurement of CK levels would likely all be done in the ED, but would not help to confirm the diagnosis of pseudoseizure as the question is specifically asking

28) A 32-year-old woman comes to the emergency department because of a 1-day history of sharp, right-sided

Ibuprofen therapy

  • Young women with recent viral illness (sore throat, runny nose) who now presents with pleuritic chest pain but otherwise normal exam and labs most consistent with a diagnosis of viral pleurisy
  • Pleurisy is basically analogous to a post-viral pericarditis, and therefore is also managed with an NSAID
  • Key idea: Pleuritic chest pain –> Pneumothorax, Pleurisy, Pericarditis, Pulmonary embolism
  • Key idea: Recurrent ulcers + Pleurisy may also be pointing to a diagnosis of pleuritis in the setting of lupus

29) A 30-year-old nulligravid woman, who is a military veteran, comes to the office because she has not had a menstrual period…

Hypothalamic hypogonadism

  • Young woman with previously normal periods who has been anovulatory for past year while deployed in Middle East (likely a stressful environment) found to have a normal physical exam with low levels of LH and FSH most concerning for hypothalamic hypogonadism
  • Key idea: Hypothalamic hypogonadism often seen in setting of physical stress (gymnast, marathon runner, etc.) or emotional stress (trauma, medical resident, etc.)
  • Turner syndrome –> Would not have had history of previously normal periods (menopause before menarche), short stature, shield chest, etc.
  • PCOS –> Overweight patient with increased LH:FSH ratio (often >3:1)
  • Premature ovarian failure –> As with menopause patients, would expect an increased FSH level because of lack of negative feedback by estrogen; most commonly seen in patient with past history of radiation therapy, patient with tobacco use or in individual with family member with Fragile X syndrome
  • Prolactinoma –> Would also lead to low LH/FSH, but would expect increased prolactin + bitemporal hemianopsia + galactorrhea

30) A 45-year-old woman comes to the office because she has not had a menstrual period for 1 year. Menses previously occurred…

Follicle-stimulating hormone

  • 45 year old woman with significant smoking history who has not had a menstrual period for 1 year with a normal BMI and a normal physical exam, most concerning for slightly premature menopause
  • Key idea: Risk factors for premature menopause includes previous history of radiation and tobacco use (https://www.rbmojournal.com/article/S1472-6483(10)00103-3/pdf#:~:text=Cigarette%20smoking%20is%20one%20such,for%20Reproductive%20Medicine%2C%202004)
  • Key idea: Patients with menopause –> Decreased estrogen production by ovary –> Decreased negative feedback on hypothalamus and pituitary –> Increased FSH
  • Beta-hCG –> pregnancy would not lead to absent periods for 1 year

31) A 2-month-old girl is brought to the office by her mother for a routine examination. Newborn screening electrophoresis showed hemoglobin FS…

Oral prophylactic penicillin therapy

  • Infant with FS result on screening electophoresis –> high-risk patient has sickle cell disease –> Should receive daily penicillin until 5 years of age + Vaccination against Strep Pneumo
  • Key idea: Sickle cell disease –> Functional asplenia due to repeated vascular insults to spleen –> Increased risk of encapsulated bacterial infection (SHiN: Strep pneumo, H influenzae, Neisseria)
  • Key idea: Most infants with FS screening have sickle cell disease (homozygous SS), but currently still have fetal hemoglobin circulating which is reason for FS result
  • Only time aspirin should be given to child is in setting of Kawasaki disease, in which case patient should receive high-dose aspirin (want anti-inflammatory effects > anti-platelet effects)

32) A 3-week-old boy is brought to the physician by his mother because of a 5-day history of yellow eyes and skin and white stools…

Ultrasonography of the abdomen

  • Previously healthy 3-week old boy presenting with jaundice, pale stools, and direct hyperbilirubinemia with severely elevated AST/ALT most concerning for biliary atresia –> Gold-standard diagnostic test is intraoperative cholangiogram, but the INITIAL diagnostic test should be a RUQ ultrasound
  • Key idea: Classic presentation of biliary atresia is asymptomatic infant who between 1-8 weeks of life will develop direct hyperbilirubinemia (due to impaired flow of bile to duodenum) with pale stools (no bile making it to small intestine) and dark urine (conjugated bilirubin is water-soluble and thereby excreted in urine)
  • Treatment: Surgery (Hepatoportoenterostomy or liver transplant)
  • General test-taking strategy: When asked to provide best initial diagnostic test, often a non-invasive answer is the correct answer (helps rule out EGD and Colonoscopy in this case)
  • Air-contrast enema –> Therapeutic and Diagnostic for Intusussception (Toddler presenting with recurrent episodes of abdominal pain and bloody stools that improve when drawing knees to chest)

33) A 52-year-old woman comes to the physician for a follow-up examination. She received the diagnosis of hypertension…

Discontinuation of lisinopril

  • Middle-aged woman recently started on an ACE inhibitor who presents with constant non-productive cough most likely due to side effect from ACE inhibitor –> Treat by discontinuing medication
  • Common causes of subacute/chronic cough: (1) ACE inhibitors (treat by stopping ACE inhibitor) (2) Asthma (get PFTs) (3) GERD (empiric PPI) (4) COPD (PFTs and imaging) (5) Upper airway cough syndrome (empiric anti-histamine medication)
  • Key idea: If you suspect one of the above causes of subacute/chronic cough but it doesn’t resolve with empiric treatment –> Chest x-ray

34) A 27-year-old primigravid woman at 41 weeks’ gestation is admitted to the hospital for induction of labor. She has received…

Neonatal respiratory depression

  • Side effects of intrapartum opiates on mother: Altered mental status, GI symptoms (constipation, etc.)
  • Side effects of intrapartum opiates on baby: Drowsy, respiratory depression, bradycardia, trouble breastfeeding early on
  • Key idea: In adults opiates lead to pinpoint pupils, respiratory depression, altered mental status, constipation, etc.
  • https://www.acog.org/womens-health/faqs/medications-for-pain-relief-during-labor-and-delivery#:~:text=Side%20effects%20are%20minor%20and,first%20few%20hours%20after%20birth.

35) A 7-year-old girl is brought to the clinic by her father because of behavior problems at school since she started second grade…

Polysomnography

  • Young girl presenting with behavior problems, snoring at night and bedwetting with enlarged tonsils on physical exam, concerning for obstructive sleep apnea
  • Key idea: Bedwetting can sometimes be caused by obstructive sleep apnea (https://www.mayoclinic.org/diseases-conditions/bed-wetting/symptoms-causes/syc-20366685)
  • Key idea: Polysomnography is the gold-standard for virtually all sleep conditions (obstructive sleep apnea, Narcolepsy, REM sleep behavior disorder, etc.)
  • No findings concerning for brain cancer (asymmetric neurological symptoms, headache and nausea/vomiting worse upon waking, etc.), which would warrant brain MRI

36) A 52-year-old man comes to the emergency department because of increasingly severe pain and swelling of his right foot…

Immobilization of the right foot in a posterior splint

  • Middle-aged man with type 2 diabetes with signs of peripheral vascular disease and diabetic neuropathy who tripped over a rug and now presents with dull pain and redness of the foot/ankle with foot x-rays consistent with a Charcot joint/foot (see image below)
  • Key idea: Charcot joint/foot is a condition primarily seen in diabetic patients with neuropathy in which there is a vicious cycle in which patients injure their foot –> don’t feel injury so continue to use it normally –> increased foot injuries/inflammation –> Still don’t feel –> Continued injury/inflammation in foot –> etc.
  • Key idea: Medical treatment of Charcot foot/joint aimed at offloading the foot and preventing further fractures, which can be done in this case with splinting
  • Key idea: In a patient with poorly controlled diabetes, would likely try conservative therapy >> Surgery of the foot because they are at high-risk for poor wound healing and post-op infections that could result in foot amputation
  • Negative Homans sign –> Rules out PE (at least on the NBME, not in real life)
  • Measurement of serum uric acid –> Gout (Arthritis of big toe or knee triggered by alcohol/meat consumption, surgery, etc.)
https://care.diabetesjournals.org/content/34/9/2123#:~:text=Charcot%20neuropathic%20osteoarthropathy%20(CN)%2C,inflammation%20in%20the%20earliest%20phase.

37) An 8-year-old boy is brought to the emergency department (ED) by ambulance 45 minutes after he was struck by a car…

Proceed with the blood transfusion while obtaining an emergency telephone court order

  • Key idea: Parents CANNOT refuse life-saving treatment for their child, regardless of the rationale
  • If the scenario had been an 18+ year old individual whose religious beliefs meant they refuse all blood products, then the correct answer would have been proceed with operation while abiding by the patient’s wishes

38) A 77-year-old man comes to the office as a new patient. He has a history of gout and surgical repair of a hernia. He has…

Aortic stenosis

  • Elderly man with hypertension with a systolic ejection murmur loudest in the aortic area (right 2nd intercostal space) with radiation to carotids that increases in intensity with increased preload (upon squatting) and decreases in intensity upon handgrip, most consistent with aortic stenosis
  • Key tip: On auscultation questions, the carotids should be quiet so in this case the radiation of the murmur to the carotids is an important clue for aortic stenosis
  • Key tip: I found it difficult to hear the S1 and S2 heart sounds, but I knew that it was a systolic murmur because the period of silence was longer than the period with the murmur and diastole is longer than systole
  • Handgrip –> Increased arteriolar vasoconstriction –> Increased afterload –> Decreased intensity of forward-flowing murmurs (aortic stenosis, mitral stenosis, HOCM, etc.) and Increased intensity of backward-flowing murmurs (mitral regurgitation, VSD, etc.)
  • Squatting –> Heart closer to legs –> Increased preload –> Increased intensity of all murmurs EXCEPT for HOCM and Mitral valve prolapse (reasoning is that for most murmurs increased preload –> increased blood flowing past messed up valves –> louder murmurs, but for HOCM and Mitral valve prolapse increased preload –> stretching of left ventricle –> decreased outflow obstruction in setting of HOCM and chordae tendinae more taut in setting of MVP)
  • Valsalva –> Increased intra-abdominal pressure –> Compression of inferior vena cava –> decreased preload –> Decreased intensity of all murmurs except for HOCM and MVP

39) A 43-year-old woman comes to the office for a follow-up examination 1 week after she was discharged from the hospital following…

Diltiazem

  • Young woman who was recently hospitalized for crushing chest pain resolving with nitroglycerin with ST-depressions on ECG and a clean cardiac catheterization who continues to have short unprovoked episodes of chest pain most consistent with Prinzmetal’s variant angina
  • Key idea: Prinzmetal’s angina should be treated with calcium channel blockers or nitrates
  • Key idea: Prinzmetal’s angina more commonly seen in young women who smoke tobacco
  • https://rarediseases.info.nih.gov/diseases/7465/prinzmetals-variant-angina#:~:text=Prinzmetal’s%20variant%20angina%20(PVA)%20is,physical%20exertion%20or%20emotional%20stress.

40) A 67-year-old man is brought to the emergency department by his wife 1 hour after he fainted while getting the morning newspaper…

Mobitz type II (second-degree) atrioventricular block

  • Elderly man with recurrent episodes of fainting/dizziness with an otherwise normal exam but an ECG consistent with second-degree AV block
  • Key tip: Can be difficult to differentiate between 2nd degree type 1 and 2nd degree type 2 AV block, but one important difference is 2nd degree type 1 leads to regularly irregular pattern (3 QRS’s followed by dropped beat followed by 3 QRS’s, etc.), whereas 2nd degree type 2 leads to an irregularly irregular pattern
  • 1st degree AV block: PR interval is prolonged (>200 ms or 1 big box) but there is a QRS after every p wave (no dropped beats)
  • 2nd degree type 1 AV block: PR interval becomes increasingly prolonged until a QRS is dropped, leading to a regularly irregular pattern
  • 2nd degree type 2 AV block: PR interval remains consistently prolonged with randomly dropped beats leading to an irregularly irregular pattern
  • 3rd degree AV block: Atria and ventricles are beating independently, such that p-waves march out at a rate ~60-100 beats/second and QRS complexes march out at a rate of ~30-40 beats/second
  • Key idea: 2nd degree type 2 AV block and 3rd degree AV block are due to pathology distal to the AV node and therefore often require treatment with a pacemaker

Block 3:

1) A 35-year-old woman is brought to the emergency department by police after they found her wandering the streets and appearing intoxicated…

Alcohol use disorder

  • Mnemonic to remember suicide risk is SAD PERSONS (Sex (males), Age (<19 or >45), Depression, Previous suicide attempt, Excess alcohol/drugs, Rationale thinking loss (psychosis), Social support lacking, Organized plan, No spouse, Sickness
  • Borderline personality disorder associated with self-harming behaviors, but not dependent personality disorder

2) A 76-year-old woman, who is receiving home hospice care because of end-stage metastatic lung cancer diagnosed 1 month ago…

Administer IV morphine

  • Elderly woman on hospice for lung cancer who is having shortness of breathe and hypoxemia despite 100% FiO2 –> because she is on hospice, we should prioritize comfort and a common practice in patients with shortness of breathe is use of IV morphine because it can decrease respiratory drive and the perception of shortness of breathe, so it makes patients feel more comfortable
  • Key idea: While one could argue that use of IV morphine in these patients may be hastening their death, the benefits of improving their comfort outweigh the risk of having them live slightly longer but with more pain/discomfort
  • Key idea: Just because a patient is on hospice does not mean that we do nothing and offer them no interventions, it’s just that the interventions are geared towards improving comfort rather than treating/curing disease

3) A 42-year-old woman comes to the physician because of a 1-month history of moderate constant abdominal pain. She was…

Medullary thyroid carcinoma

  • Young woman with strong family history of hyperparathyroidism and adrenal tumor who presents with severe, refractory hypertension and increased urine metanephrines (consistent with a pheochromocytoma) and significant hypercalcemia complicated by abdominal pain and renal calculi (consistent with hypercalcemia possibly due to parathyroid hyperplasia) which is most consistent with one of the Multiple endocrine neoplasia syndromes, particularly MEN 2A –> increased risk of medullary thyroid cancer
  • MEN 1 –> 3 P’s –> Pituitary tumors, Parathyroid adenoma, Pancreatic tumor (Zollinger-Ellison, VIPomas, etc.)
  • MEN 2A –> 2 P’s and 1 M –> Parathyroid adenoma, Pheochromocytoma, Medullary thyroid cancer
  • MEN 2B –> 1 P and 2 M’s –> Pheochromocytoma, Medullary thyroid cancer, Mucosal neuromas
  • Key idea: Remember that PArathyroid adenoma is in MEN 1 and MEN 2A whereas PHeochromocytoma is in MEN 2A and 2B because PA > PH alphabetically

4) A 26-year-old woman with common variable immunodeficiency comes to the office because of a 2-month history of night sweats…

Lymph node biopsy

  • Young woman with CVID and a chronic history of recurrent infections who presents with B symptoms (weight loss, night sweats) and continuous lymphadenopathy concerning for lymphoma which should be worked up with lymph node biopsy
  • Patients with CVID have an increased risk of lymphoma through multiple mechanisms, but important to remember the general principle that immunodeficiencies are often associated with increased risk of infection (https://pubmed.ncbi.nlm.nih.gov/18536576/)
  • Key idea: Marginal zone lymphomas can also often be seen in the setting of repeated/chronic inflammation (Sjogren syndrome –> Parotid inflammation –> Lymphoma // Chronic autoimmune gastritis –> Stomach inflammation –> MALT Lymphoma)
  • Note: Epitrochlear lymphadenopathy (lymph nodes in upper arm) often makes me think of syphilis infection, but the presentation not consistent in this case

5) A 9-month-old boy is brought to the office by his mother because of a 6-hour history of persistent fussiness and decreased appetite…

Skeletal survey

  • Infant with recent illness brought to the office due to 6 hours of inconsolable crying found to have a rash consistent with 2nd degree burns on the buttocks (red and painful with blisters) and a mother who is denying knowledge of what happened, which should prompt concern for child abuse
  • Situations to consider child abuse: (1) Bruises/fractures at various stages of healing (2) Injury story not compatible with development (8 month old was running, fell down and broke his arm) (3) Retinal hemorrhages (4) Well-demarcated burns showing classic pattern (see below)
  • Key idea: Child with signs of abuse should receive skeletal survey (look for current and past fractures), CT scan of the head (look for subdural hematoma due to “shaken baby” syndrome) and Fundoscopic examination (Retinal hemorrhages quite specific for “shaken baby”)
  • Pet lizard –> Risk factor for non-typhi Salmonella –> Febrile GI illness
  • Anti-Saccharomyces cerevisiae antibody –> Crohn’s disease –> Chronic history of abdominal pain, bloody diarrhea, skin findings, etc.
  • Nitroblue tetrazolium testing –> Chronic granulomatous disease –> Increased risk of Catalase positive organisms, often presents with recurrent Staph skin infections
https://www.memorangapp.com/flashcards/37860/Child+Abuse,+SIDS,+Poisonings/

6) A 62-year-old woman is admitted to the hospital because of an 8-hour history of severe right upper quadrant pain…

Ultrasonography of the right upper abdominal quadrant

  • Middle-aged obese woman with history of recurrent self-resolving episodes of RUQ pain after meals (consistent with biliary colic) who now presents with a longer-lasting, more severe episode of post-prandial RUQ pain + fever + near-hypertension and bradycardia + signs of peritonitis (rebound, guarding) + labs consistent with pancreatic/biliary/liver disease (elevated amylase/lipase and low calcium points to possible pancreatitis, elevated AST/ALT with leukocytosis concerning for cholecystitis vs acute cholangitis) who should be worked up further with RUQ ultrasound to look for biliary disease
  • Key idea: If a patient comes in with obvious signs of pancreatitis (elevated amylase/lipase, abdominal pain radiating to the back) and they don’t use alcohol, then next best diagnostic step would also be a RUQ ultrasound to look for gallstones –> If gallstones present then patient should have cholecystectomy within a few days and if absent then patient needs to be worked-up for other causes of pancreatitis (hypercalcemia, hypertriglyceridemia, etc.)
  • Key idea: ERCP is indicated for acute cholangitis and pancreatic cyst/mass, but importantly is NOT indicated in setting of gallstone pancreatitis unless patient is suspected of having concomitant acute cholangitis because often gallstone pancreatitis is not caused by a stone stuck in the biliary tree but rather by spasm of the main pancreatic duct where it meets the common bile duct due to irritation from a passing stone

7) A 28-year-old woman, gravida 2, para 1, at 39 weeks’ gestation with an intrauterine pregnancy, is admitted to the hospital for induction…

Umbilical cord prolapse

  • Young woman with complicated pregnancy (polyhydramnios, gestational diabetes) who has artificial rupture of membranes followed by a prolonged deceleration (on order of minutes), concerning for umbilical cord prolapse
  • Key idea: Umbilical cord prolapse should be suspected if a patient experiences membrane rupture (spontaneous or artificial) and then shortly after develops abnormal fetal heart tracings, with common ones being seen in umbilical cord prolapse being fetal bradycardia, variable decelerations and prolonged decelerations > 1 minute)
  • Risk factors for umbilical cord prolapse: Breech presentation, artificial rupture of membranes, polyhydramnios
  • Management of umbilical cord prolapse: Elevate the fetal head and perform a C-section (NOT appropriate to replace the cord or to allow the patient to continue to labor)
  • Early deceleration –> Deceleration mirrors contraction –> Sign of fetal head compression –> Benign finding
  • Variable deceleration –> Unassociated with contractions, steep/abrupt deceleration and recovery –> Sign of umbilical cord compression –> Most concerning if recurrent and with loss of variability
  • Late deceleration –> Deceleration occurs following contraction (nadir does not mirror peak of contraction) –> Uteroplacental insufficiency –> Most concerning if recurrent and with loss of variability; often most ominous of the deceleration patterns

8) A 63-year-old man with chronic obstructive pulmonary disease comes to the urgent treatment center because of a 4-day history of worsening…

Titrated oxygen therapy

  • Elderly man with COPD and significant past smoking history who presents with acute worsening dyspnea with hypoxemia (85% O2 sat) on room air and an increased A-a gradient most consistent with a COPD exacerbation
  • PaO2 = 48 (based on ABG)
  • PAO2 = 150 – (PaCO2 / 0.8) = 150 – (55/0.8) = 150-69 = 81
  • Normal A-a gradient less than (Age/4) = (63/4) = 16
  • A-a gradient = 81 – 48 = 33 > 16 –> Increased A-a gradient concerning for V/Q mismatch
  • Key idea: Acute COPD exacerbation should be treated with inhaled bronchodilators (beta-2 agonists and anticholinergics) + Systemic steroids +/- Supplemental oxygen (with goal between 88 and 92%) +/- Oseltamivir (if evidence of influenza) +/- Antibiotics (if 2/3 of following: increased dyspnea, increased cough, increased/colored sputum)
  • Key idea: Goal O2 saturation for COPD patients NOT in the high 90’s because these patients often walk around with baseline CO2 retention such that their respiratory drive is desensitized to CO2 levels and relies primarily on O2 levels, so if you push their O2 level too high it can lead to decreased respiratory drive

9) A 49-year-old man returns to the office for follow-up 2 weeks after he was evaluated for a 4-month history of intermittent diarrhea associated…

Adenocarcinoma of the colon

  • Middle-aged man with intermittent episodes of diarrhea with blood/mucus with colonoscopy showing rectal and sigmoid colon disease with ileal sparing and erythema nodosum, most consistent with ulcerative colitis
  • Key idea: Crohn’s disease can lead to patchy disease in any part of GI tract from mouth to anus but tends to spare the rectum, whereas UC leads to continuous disease that starts in the rectum and typically also involves the colon, but spares the terminal ileum
  • Key idea: Patients with IBD (UC > Crohn’s) have much higher risk of colon cancer and should begin colon cancer screening 8-10 years after diagnosis
  • Key idea: Both forms of IBD (Crohn’s and UC) are associated with skin disease (pyoderma gangrnosum, erythema nodosum), joint disease (inflammatory arthritis), eye disease (uveitis) and oral ulcers
  • Crohn’s disease is specifically associated with calcium oxalate kidney stones (terminal ileal disease –> Impaired fat absorption –> Intra-intestinal calcium binds to fat instead of oxalate –> More free oxalate absorbed from the gut)
  • Ulcerative colitis is specifically associated with primary sclerosing cholangitis (leads to direct hyperbilirubinemia)
  • Differential of erythema nodosum: Sarcoidosis, Inflammatory bowel disease, tuberculosis, Behcet’s syndrome, fungal infections (cocci, histo, etc.)

10) A 27-year-old man is examined in the intensive care unit 1 day after he sustained a closed head injury, liver laceration, and pelvic…

Decreased uptake of glucose by insulin-sensitive tissues

  • Key idea: Insulin resistance is a fundamental response to injury/stress and is a well-known phenomenon in the post-op and post-trauma patient
  • https://academic.oup.com/jcem/article/95/9/4217/2835313

11) A 72-year-old woman is evaluated in the acute rehabilitation facility where she has been receiving physical therapy since sustaining…

Discharge the patient with a plan for home health care and physical therapy

  • Key idea: This option meets the needs/goals of the patient while also ensuring they receive an appropriate level of care
  • B seems too harsh and we should not simply withdraw treatment from the patient, C doesn’t account for patient autonomy and D seems incorrect because based on the vignette her decision-making capacity seems to be intact

12) A 46-year-old woman comes to the office for an annual health maintenance examination. She reports a 6-week history…

Hyperaldosteronism

  • Middle-aged woman with relatively normal history/physical except for persistent hypertension, hypokalemia and borderline metabolic alkalosis, most concerning for hyperaldosteronism
  • Key idea: Increased aldosterone levels, whether due to Conn syndrome or causes of RAAS activation (hypovolemia, decreased effective circulating volume due to CHF exacerbation, renal artery stenosis, etc.), leads to hypokalemia and metabolic alkalosis because Aldosterone –> Increased ENaC channels –> Increased Na+ reabsorption in exchange for H+ and K+ secretion/excretion in order to maintain electrical neutrality
  • Cushing syndrome –> Same electrolyte abnormalities with increased central adiposity, buffalo hump, etc.
  • Fibromuscular dysplasia –> Same electrolyte abnormalities + Abdominal bruit +/- neurologic signs associated with fibromuscular dysplasia (dizziness, syncope, etc.)
  • Hypothyroidism –> Weight gain, bradycardia, cold tolerance, delayed relaxation of reflexes
  • OSA –> Overweight patient with history of snoring and excessive tiredness/headache in the morning

13) A 50-year-old woman is brought to the emergency department because of a 3-day history of severely painful lesions over her right leg…

Necrotizing fasciitis

  • Middle-aged woman with type 2 diabetes presenting with fever and a lower extremity lesion that is exquisitly tender, lacks sharp margins and has bullae with purplish discharge most consistent with necrotizing fasciitis
  • Signs pointing to necrotizing fasciitis over cellulitis: (1) Pain out of proportion to examine (extreme pain > tenderness) (2) Rapid progression of symptoms (3) Severe systematic features (fever, chills, hypotension) (4) Crepitus or free air (5) Bullae (6) Purulent, cloudy discharge (7) Violaceous appearance of lesion (8) Paresthesia/Anesthesia at edge of lesion
  • Key idea: Patients with suspected necrotizing fasciitis SHOULD GO DIRECTLY TO OR!!!
  • Key idea: Diabetic patients predisposed to necrotizing fasciitis of the perineal agent called Fournier gangene

14) A 37-year-old woman comes to the office for a routine health maintenance examination. For the past 6 years, she has worked…

Using a respirator at work

  • Key idea: Inhalation of chlorine gas can lead to lung injury over time (https://www.ncbi.nlm.nih.gov/pmc/articles/PMC3136961/), and when trying to approach the problem try to implement Primary disease prevention whenever possible because it is better to prevent disease than to screen/treat disease
  • Primary disease prevention: Prevent disease before it occurs (vaccination, wearing a respirator)
  • Second disease prevention: Screen for disease and manage asymptomatic disease (Monthly pulmonary function tests)
  • Tertiary disease prevention: Treatment of disease (Corticosteroid therapy)

15) A 57-year-old man with alcohol use disorder comes to the emergency department because of a 3-day history of worsening…

Spironolactone

  • Middle-aged man with history of alcoholic cirrhosis who is still drinking and presents with signs of decompensated cirrhosis (distended abdomen, positive fluid wave, etc.) who should therefore be treated with spironolactone because it is the diuretic of choice for cirrhotic ascites
  • Unfortunately cannot easily reason through this question, just an important fact to commit to memory that has been shown time and time again through research trials (https://acphospitalist.org/archives/2014/02/expert.htm)
  • Note: Could also argue to begin propranolol both to target hypertension and to decrease risk of variceal hemorrhage, but in this case they are mainly focused on managing ascites in setting of cirrhosis

16) A 48-year-old man is admitted to the hospital 2 hours after the onset of severe abdominal pain, nausea, and vomiting. The pain radiates…

Gemfibrozil therapy

  • Middle-aged man without significant alcohol history presenting with abdominal pain radiating to the back and elevated amylase found to have a triglyceride level in the 1000’s, most consistent with pancreatitis due to hypertriglyceridemia
  • Key idea: Most of the time statins are the best lipid-lowering agent, with one key exception being the setting of hypertriglyceridemia-associated pancreatitis which should be treated with a fibrate because they lead to the most profound drop in triglyceride levels
  • Diagnostic criteria for acute pancreatitis is at least 2/3 of following: (1) Back pain radiating to the back (clinical) (2) Amylase and/or Lipase 3X upper limit of normal (labs) (3) Imaging findings compatible with pancreatitis (imaging)
  • 2 most common causes of pancreatitis are Gallstones and Ethanol, with a mnemonic to remember most causes being I GET SMASHED: Idiopathic, Gallstones, Ethanol, Trauma (to the abdomen), Scorpion sting, Mumps, Autoimmune, Steroids, Hypercalcemia/Hypertriglyceridemia, Drugs (sulfa, HIV drugs)

17) A 57-year-old man is admitted to the hospital 30 minutes after he was found wandering in the streets. Fingerstick blood glucose…

Vitamin B1 (thiamine)

  • Middle-aged man with signs of cirrhosis (conjunctival icterus, fluid wave, palmar erythema, etc.) brought to the hospital with altered mental status and ataxia concerning either for alcohol intoxication versus Wernicke encephalopathy
  • Key idea: Patient using substances who is brought to the ED with altered mental status should almost always receive Naloxone, Thiamine and glucose, and importantly, as discussed in a previous question, they should receive thiamine BEFORE glucose because glucose can lead to shifts of water-soluble vitamins into cells/tissue that can push a patient into Wernicke’s encephalopathy
  • Key idea: Wernicke encephalopathy classically leads to confusion, ataxia, ophthalmoplegia
  • SUPER KEY: On the NBME, when you see ALCOHOLIC you should translate that to MALNUTRITION (Wernicke, folate deficiency, Scurvy, etc.)

18) Three days after an operation for uterine cancer, a hospitalized 67-year-old woman develops edema of the right lower extremity…

Develop an institutional policy with standardized orders for DVT prophylaxis

  • Key idea: The answer to these questions is often standardization of a human error-prone task and/or intervention that improves communication between healthcare team members
  • Key idea: The answer to these questions is RARELY educational policies (excludes A)
  • Early ambulation and Anticoagulation in the early post-op period, while good policies for most patients, may not be appropriate for ALL patients and therefore should NOT be applied universally

19) A clinical trial is performed to assess the efficacy of aspirin in decreasing the risk of myocardial infarction. Twenty-two thousand…

Absolute risk reduction is 1%

  • Absolute risk reduction = Risk in placebo – Risk in aspirin = 2.2 – 1.2 = 1.0%
  • Relative risk reduction = 1 – Relative Risk = 1 – (Risk in aspirin / Risk in placebo) = 1 – (1.2 / 2.2) = 1 – (6/11) = 5/11 = 0.45
  • Key idea: Number needed to treat = 1 / ABSOLUTE risk reduction = 1 / 0.01 = 100

20) An 18-year-old man is being prepared to undergo left temporal craniotomy for meningioma. General anesthesia has been…

Ask the surgeon to continue the operation only after the discrepancy is resolved

  • Definitely would want to verify the operation site before beginning (narrowing it down to Answers B, C and E), with B being incorrect because it is not the circulating nurse’s responsibility to read MRIs and E is incorrect because it would be inappropriate to call the patient’s parents to confirm because (1) They may not be in the waiting room (2) The patient is an adult so his medical records are confidential (3) It would reflect poorly on the surgical team that they don’t know which side is correct

21) A 52-year-old man is brought to the emergency department because of a 2-day history of severe, progressive shortness of…

Application of positive end-expiratory pressure

  • Middle-aged man with smoking history and likely COPD given baseline history presents in respiratory distress with significant hypoxemia despite mechanical ventilation with x-ray findings of diffuse bilateral alveolar infiltrates most concerning for Acute Respiratory Distress Syndrome (ARDS)
  • Patients with ARDS are typically managed with the ARDSnet Ventilation Strategy, which basically advocates for high positive end-expiratory pressure (PEEP) with low tidal volumes (https://litfl.com/ardsnet-ventilation-strategy/)
  • Key idea: When a patient is mechanically ventilated, oxygenation is based upon FiO2 and PEEP, so in this patient who continues to have hypoxemia despite receiving 100% oxygen, the only course of action is to change the current ventilation settings by increasing PEEP

22) A 2-day-old female newborn is examined in the nursery. She was born at term to a 24-year-old primigravid woman following…

Observation only

  • 2-day old newborn with normal/healthy prenatal course, post-delivery course and vitals who has a harsh, continuous machine-like murmur most consistent with a patent ductus arteriosus
  • Key idea: Normal for PDA to remain open for first 2-3 days of life and actually the murmur will likely get louder before it resolves completely (smaller holes –> more turbulent blood flow –> louder murmur)
  • Key idea: Can tell that the murmur is continuous (i.e. involves both systole and diastole), because there is no noticeable pause in the sound of the murmur

23) A 65-year-old man with mild hypertension comes to the veterans’ administration outpatient clinic because he is…

Duplex ultrasonography of the abdomen

  • Key idea: USPSTF recommendations are that MALES 65-75 years of age with ANY SMOKING HISTORY should receive a one-time duplex ultrasound to screen for abdominal aortic aneurysm
  • Important to contrast with lung cancer screening, which should occur in patients of either sex but only if they are between 55 and 80 years of age and have a smoking history of at least 30 pack-years and they have smoked within the past 15 years
  • https://www.uspreventiveservicestaskforce.org/uspstf/recommendation/abdominal-aortic-aneurysm-screening#:~:text=The%20USPSTF%20recommends%201%2Dtime,years%20who%20have%20ever%20smoked.&text=The%20USPSTF%20recommends%20against%20routine,no%20family%20history%20of%20AAA.

24) A 57-year-old man comes to the physician because of a 1-month history of fatigue and moderate low back pain. He also has had…

Serum protein electrophoresis

  • Middle-aged man presenting with fatigue, back pain, anemia, hypercalcemia and mild creatinine elevation most concerning for multiple myeloma which should be worked up with serum protein electrophoresis (SPEP)
  • Key idea: Most cases of renal dysfunction lead to HYPOcalcemia (increased phosphate binds calcium, vitamin D activation in kidney disrupted), so if you see renal dysfunction + hypercalcemia you should heavily consider multiple myeloma

25) A 38-year-old woman, gravida 1, para 1, comes to the office because she has been unable to conceive for the past year…

Osteoporosis

  • 38 year old woman of normal weight who was able to conceive a child 6 years ago who now presents with difficulty conceiving in setting of irregular menses found to have a normal prolactin and an increased FSH most concerning for premature ovarian failure (menopause before 40 years of age)
  • Premature ovarian failure –> Decreased lifetime estrogen exposure –> Increased risk of osteoporosis (we know that menopause –> osteoporosis, so it makes sense that early menopause would be even worse in terms of osteoporosis risk)
  • Typical risk factors for premature ovarian failure: History of radiation, family history of Fragile X syndrome
  • Key idea: In setting of oligo/amenorrhea, if FSH is high it tells you that hypothalamus and pituitary are functioning but ovaries can’t be stimulated (menopause, primary ovarian failure), whereas if FSH is low then you know the problem lies in the pituitary (prolactinoma, Sheehan syndrome) or hypothalamus (functional hypothalamic amenorrhea)
  • PCOS (heavy woman with signs of hyperandrogenism) –> Endometrial hyperplasia/cancer

26) A 27-year-old man comes to the physician because of a 1-month history of progressive fatigue and episodes of moderate…

von Willebrand disease

  • Young man who has a sister with bleeding problems who himself has recurrent nosebleeds refractory to treatment, anemia and a prolonged aPTT most consistent with von Willebrand disease
  • Key idea: von Willebrand disease is primarily a qualitative platelet disorder (nose bleeds, petechiae, purpura with a normal platelet count and increased bleeding time) but can also lead to a mildly increased aPTT because von Willebrand factor is a carrier protein for Factor VIII (part of intrinsic coagulation pathway)
  • Hemophilia A is X-linked so would be rare to have a sister with the disease
  • Hereditary hemorrhagic telangiectasia leads to nose bleeds, telangiectasias and, most concerning, AV fistulas (high-output heart failure)

27) A previously healthy 57-year-old woman comes to the physician because of a 2-month history of mild right lower abdominal…

Right hemicolectomy

  • Middle-aged woman with a 2-month history of fatigue and abdominal pain found to have anemia and a moderately differentiated adenocarcinoma in the ascending colon most consistent with right-sided colon cancer
  • Key idea: Surgical resection is the treatment of choice for virtually all stages of colorectal cancer, regardless of side (left vs right)
  • https://www.cancer.org/cancer/colon-rectal-cancer/treating/colon-surgery.html

28) A 37-year-old woman comes to the office for a follow-up examination 3 weeks after testing confirmed a mass in her left breast…

Generalized anxiety disorder

  • Young woman with chronic history of anxiety that has been exacerbated by a recent cancer scare and continues to stress her out despite reassuring testing most consistent with generalized anxiety disorder
  • Key idea: Generalized anxiety disorder diagnosed in a patient with chronic symptoms (>6 months) of worries about a myriad of life issues (“Generalized”)
  • Note: Patient may have some component of illness anxiety disorder (Preoccupation with having a serious illness often despite medical evaluation and reassurance) and social anxiety disorder (exaggerated fear of embarrassment in social situation), but that wouldn’t explain her chronic history of anxiety about other issues unrelated to breast cancer

29) A 55-year-old man comes to the emergency department because of a 6-hour history of diffuse abdominal pain and subjective fever…

Cefotaxime

  • Middle-aged man with alcoholic cirrhosis who presents with diffuse abdominal pain and fever with an abdominal paracentesis showing a PMN count > 250 cells consistent with spontaneous bacterial peritonitis
  • Key idea: Main treatment of Spontaneous bacterial peritonitis is a third-generation cephalosporin such as Cefotaxime, whereas some cirrhosis patients can receive Fluoroquinolones (-floxacin) for prophylaxis
  • Key idea: Ascitic fluid with PMN > 250 cells/mm^3 is diagnostic for spontaneous bacterial peritonitis
  • Key idea: Patients with cirrhosis often don’t mount as robust of an immune response in setting of infection, so a temperature of 100 F (37.8 C) or above should make you concerned!
  • Key idea: Any time a patienty with cirrhosis presents with altered mental status, diffuse abdominal tenderness, or temperature > 100 F, your index of suspicion for spontaneous bacterial peritonitis should be high and patient should receive a Paracentesis

30) A 45-year-old man is brought to the emergency department because of moderate chest pain after a generalized tonic-clonic seizure…

Vitamin D

  • Vitamin D deficiency –> Osteomalacia/Rickets –> Weak bones + signs of excess bone deposition in response to insufficient mineralization (Frontal bossing, Rachitis rosary, Wrist widening, etc.)
  • Key idea: Many antiepileptics (including phenobarbital and carbamazepine) lead to induction of the cytochrome P-450 system, which leads to increased metabolism of vitamin D and can lead to vitamin D deficiency (seen in up to 50% of patients taking long-term anti-epileptic therapy) [https://www.ncbi.nlm.nih.gov/pmc/articles/PMC3427195/]
  • Key idea: Carbamazepine also associated with folate deficiency, but that would not affect bone health and would instead lead to a megaloblastic anemia

31) A 32-year-old man comes to the office because of a 2-day history of redness of the right eye. He has not had…

Ketorolac 0.5% solution, 1 drop in right eye, four times daily

  • Key idea: Although we use abbreviations to expedite note-writing and other tasks related to documentation, the NBME likes to emphasize that using clear language that is generally interpretable is best for avoiding mistakes or issues with communication

32) A 14-year-old girl is brought to the physician by her mother for a well-child examination. Vital signs are within normal limits…

Pubic hair development: Normal // Breast development: Normal

  • 14-year-old girl with Tanner Stage 4 breast and pubic hair development, reflecting normal development (see chart below)
  • Related note: Primary amenorrhea diagnosed in girls who have not had secondary sexual changes (breast development) by 13 or who have not had menarche by 15
  • Key idea: Normal female pubertal development often involves breast changes beginning at age 8-12 years old, followed ~1-2 years later by a growth spurt, then followed ~6 months later by menarche (first menstrual period)
First AID for the USMLE Step 1 2020, Thirtieth Edition

33) A healthy 19-year-old woman comes to the clinic requesting advice regarding contraception. Menses occur at regular 28-day…

Subdermal contraceptive implant

  • Key idea: Subdermal contraceptive implant often placed in the upper arm and can be effective for ~3-5 years!
  • Although vaginal ring (must remove and replace every month) and oral contraceptives (daily medication) are highly effective forms of contraception, they require more effort than the subdermal implant
  • Note: Condoms, vaginal diaphragm and other forms of barrier contraception are not as effective as hormonal contraception and therefore are basically never the correct answer on an NBME exam
  • Note: This patient would likely NOT be a good candidate for a Copper IUD because she has heavy flow and the copper IUD can lead to even heavier flow

34) A 56-year-old man with adenocarcinoma of the lung comes to the physician because of a 1-week history of shortness of breath with exertion…

Increased total protein concentration

  • Middle-aged man with lung cancer who recently completed an initial course of chemotherapy presents with progressive dyspnea on exertion found to have dullness to percussion and decreased breath sounds of the right lung base with CXR consistent with a right pleural effusion which is most likely secondary to malignancy –> Exudative infection
  • Light’s criteria (If at least 1 of the following conditions are true, then the effusion is exudative): (1) Pleural fluid protein: Serum protein ratio > 0.5 /// (2) Pleural fluid LDH: Serum LDH ratio > 0.6 /// (3) Pleural fluid LDH > 2/3 upper limit of normal serum LDH (often ~200)
  • EXudative effusions are due to increased vascular permeability often in the setting of inflammation (cancer, infection, autoimmune) and you can remember that they have fluid and EXtra substances (protein, cells, LDH, etc.)
  • Transudative effusions are often due to increased hydrostatic pressure (volume overload due to CHF exacerbation, CKD, etc.) or decreased oncotic pressure (nephrotic syndrome, malnutrition, etc.)
  • Empyema (walled off, purulent fluid collection secondary to infection) –> decreased glucose concentration (bacteria eat sugar), increased segmented neutrophil count

35) A previously healthy 17-year-old girl comes to the emergency department because of a 3-day history of moderate right lower quadrant pain, fever…

Tubo-ovarian abscess

  • Young woman with high-risk sexual practices (multiple partners, inconsistent condom use) who presents with acute RLQ, fever and blood-tinged discharge from the cervical os with cervical motion tenderness, most consistent with pelvic inflammatory disease complicated by a tubo-ovarian abscess (only diagnosis that can explain the cervical discharge and fever)
  • Young woman with lower quadrant abdominal pain differential: (1) Appendicitis (febrile, right-sided ONLY) // (2) Ectopic pregnancy (positive pregnancy test, potential hemodynamic instability) // (3) Ovarian torsion (reduced Doppler flow, especially seen in patients with known history of ovarian teratoma due to unbalanced nature of mass) // (4) Pelvic inflammatory disease (cervical motion tenderness, high-risk sexual practices)
  • Note: Of the above diagnoses, the only ones that classically lead to fever are appendicitis and pelvic inflammatory disease
  • Key idea: Although cervical motion tenderness most associated with pelvic inflammatory disease, it can also be seen in ectopic pregnancy and endometriosis so don’t immediately jump to PID in setting of cervical motion tenderness
  • Ectopic pregnancy –> Excluded given patient’s menstrual history (often seen in a patient who has not had a period on the order of weeks-months)

36) A 15-year-old girl is brought to the office because she has never had a menstrual period. She has no history of serious illness…

Gonadectomy

  • 15-year old girl with primary amenorrhea (no menarche by 15) with an XY karyotype, elevated testosterone and absent uterus most consistent with androgen insensitivity syndrome –> First step is removal of testicles due to high risk of malignancy (testicles at higher body temperature at higher risk of malignancy, same reason patients with Cryptorchidism have higher risk of malignancy)
  • Key idea: Primary amenorrhea diagnosed in girls who have not had secondary sexual changes (breast development) by 13 or who have not had menarche by 15
  • Key idea: Patients with androgen insensitivity syndrome have normal breast development due to aromatization of androgens –> estrogen –> breast tissue development but will have absent axillary/pubic hair because growth of hair in those areas is driven by testosterone in both men and women
  • First step in primary amenorrhea work-up: Determine if uterus is present (physical exam, imaging) because if it is ABSENT then the patient either has Mullerian agenesis (XX, normal breast and pubic/axillary hair development) or Androgen Insensitivity Syndrome (XY, absent pubic/axillary hair with normal breast development)
  • Second step in primary amenorrhea work-up: If patient with primary amenorrhea has a uterus, then next step is an FSH level because a low FSH tells you the pituitary/hypothalamus is dysfunctional (prolactinoma, hypothyroidism, functional hypothalamic amenorrhea), a normal FSH tells you they have a functional defect (imperforate hymen), and an elevated FSH tells you the ovaries are dysfunctional and can’t be stimulated (Turner syndrome, Primary ovarian failure)

37) A 50-year-old man returns to the office for a follow-up examination. During the past 2 months, he has had increased blood pressure…

Hydrochlorothiazide

  • Hydrochlorothiazide can lead to hyperuricemia and therefore should NOT be used in a patient with gout
  • Key idea: Main side effects of hydrochlorothiazide can be remembered with mnemonic hyperGLUC (hyperGlycemia, hyperLipidemia, hyperUricemia, hyperCalcemia)

38) A 23-year-old woman is brought to the emergency department by her friends 30 minutes after the sudden onset of nausea…

Reaction to food additive

  • Young woman who was eating at an Asian restaurant and developed nausea, facial flushing and chest pressure most concerning for an adverse reaction to monosodium glutamate (MSG)
  • Constellation of symptoms previously termed “Chinese Restaurant syndrome” (due to Chinese restaurants using lots of MSG), but name has now been changed to MSG Symptoms complex (https://www.healthline.com/health/chinese-restaurant-syndrome#symptoms)
  • Panic attack wouldn’t last for 30 minutes and often not associated with rash (erythema of neck)
  • Carcinoid syndrome –> Recurrent episodes of diarrhea, wheezing, flushing that can eventually lead to right-sided valvular lesions due to a serotonin-secreting small intestinal tumor that has metastasized to the liver
  • Systemic mastocytosis is a rare disorder due to accumulation of mast cells –> Episodes similar to allergic reaction (urticaria, itching, GI symptoms, etc.)
  • Note: Other random reaction associated with food is Scombroid poisoning, which leads to headache and light-headedness after eating spoiled fish

39) A 45-year-old woman returns to the office to discuss results of a fine-needle aspiration biopsy of a 0.5-cm right breast mass…

Ask the patient if she would be willing to return to the office in 2 weeks to discuss treatment options again

  • Key idea: Receiving bad news can be a disorienting/shocking experience, and therefore it would be important to speak with the patient in a few days/weeks after she has had time to fully process the news and think about her care
  • Answer A would destroy the patient-physician relationship, Answer B would lead to poor outcomes for the patient because cancer patients shouldn’t go without care for 6 months and Answer D would be inappropriate because you shouldn’t immediately refer a patient to another physician because you disagree with their choices

40) A 47-year-old woman comes to the office because of a 2-month history of progressive weakness and numbness of her right leg…

S1 radiculopathy

  • Middle aged woman with risk factors for radiculopathy (heavy lifting) who presents with weak hamstrings (knee flexion) and calves (plantarflexion), decreased Achilles reflex and decreased sensation over lateral/inferior foot, which matches most with an S1 radiculopathy
  • Mnemonic to remember lower extremity myotomes: L3 – Raise your knee (hip flexion), L4 – Kick the door (knee extension), L5 – Lift your Big Guy (Dorsiflexion and Big Toe Extension), S1 – Jump for Fun (Plantarflexion)
  • Way to remember lower extremity dermatomes: L3 covers the medial leg at the level of the groin and then the nerve roots will spiral around the leg/thigh from the medial surface (L3) to the anterior surface (L4), to the lateral surface (L5) and lastly to the posterior surface (S1)
  • Mnemonic for main lower extremity reflexes: L2-L4 – Kick the Door (patellar reflex), S1-S2 – Buckle my shoe (achilles reflex),
https://en.wikipedia.org/wiki/Dermatome_(anatomy)#/media/File:Grant_1962_663.png

We are not affiliated with the NBME, USMLE or AAMC.

The answer explanations may not be reproduced or distributed, in whole or in part, without written permission of Step Prep.